Surgery Mrcgp 2015

Published on June 2016 | Categories: Documents | Downloads: 37 | Comments: 0 | Views: 306
of 93
Download PDF   Embed   Report

Comments

Content

Question 1 of 69
A 62-year-old man presents with insomnia and lethargy. He has no other systemic symptoms of
note. Routine clinical examination reveals a palpable mass in the right lower quadrant of the
abdomen, which doesn't move with respiration and is non-pulsatile. What is the most appropriate
management?
A. Blood screen including LFTs, U&Es
B. Urgent referral to local urological service
C. Ultrasound abdomen
D. Urgent referral to local colorectal service
E. Routine referral to general surgical clinic

Colorectal cancer: referral guidelines
NICE recommend the following patients are referred urgently (i.e. within 2 weeks) to colorectal
services for investigation:







patients > 40 years old, reporting rectal bleeding with a change of bowel habit towards looser
stools and/or increased stool frequency persisting for 6 weeks or more
patients > 60 years old, with rectal bleeding persisting for 6 weeks or more without a change
in bowel habit and without anal symptoms
patients > 60 years old, with a change in bowel habit to looser stools and/or more frequent
stools persisting for 6 weeks or more without rectal bleeding
any patient presenting with a right lower abdominal mass consistent with involvement of the
large bowel
any patient with a palpable rectal mass
unexplained iron deficiency anaemia in men or non-menstruating women (Hb < 11 g/dl in
men, < 10 g/dl in women)

Question 2 of 69
The mother of a 2-month-old boy comes to surgery as she has noticed a soft lump in his right groin
area. There is no antenatal or postnatal history of note. He is breast feeding well and is opening his
bowels regularly. On examination you note a 1 cm swelling in the right inguinal region which is
reducible and disappears on laying him flat. Scrotal examination is normal. What is the most
appropriate action?
A. Refer to paediatric surgery
B. Refer to orthotics for fitting of a Pavlik harness
C. Reassure mother + ask her to return if not resolved by 6 months
D. Reassure mother + ask her to return if not resolved by 12 months
E. Reassure mother + ask her to return if not resolved by 2 years

Congenital inguinal hernias have a high rate of complications and should be repaired promptly once
identified.
Abdominal wall hernias
The classical surgical definition of a hernia is the protrusion of an organ or the fascia of an organ
through the wall of the cavity that normally contains it.
Risk factors for abdominal wall hernias include:





obesity
ascites
increasing age
surgical wounds

Features





palpable lump
cough impulse
pain
obstruction: more common in femoral hernias



strangulation: may compromise the bowel blood supply leading to infarction

Types of abdominal wall hernias:

Type of hernia

Details

Inguinal hernia

Inguinal hernias account for 75% of abdominal wall hernias. Around 95% of patients are male;
men have around a 25% lifetime risk of developing an inguinal hernia.
Above and medial to pubic tubercle
Strangulation is rare

Femoral hernia

Below and lateral to the pubic tubercle
More common in women, particularly multiparous ones
High risk of obstruction and strangulation
Surgical repair is required

Umbilical hernia

Symmetrical bulge under the umbilicus

Paraumbilical
hernia

Asymmetrical bulge - half the sac is covered by skin of the abdomen directly above or below
the umbilicus

Epigastric hernia

Lump in the midline between umbilicus and the xiphisternum
Most common in men aged 20-30 years

Incisional hernia

May occur in up to 10% of abdominal operations

Spigelian hernia

Also known as lateral ventral hernia
Rare and seen in older patients
A hernia through the spigelian fascia (the aponeurotic layer between the rectus abdominis
muscle medially and the semilunar line laterally)

Obturator hernia

A hernia which passes through the obturator foramen. More common in females and typical
presents with bowel obstruction

Richter hernia

A rare type of hernia where only the antimesenteric border of the bowel herniates through the
fascial defect

Abdominal wall hernias in children:

Type of hernia

Details

Congenital inguinal hernia

Indirect hernias resulting from a patent processus vaginalis
Occur in around 1% of term babies. More common in premature babies and boys
60% are right sided, 10% are bilaterally
Should be surgically repaired soon after diagnosis as at risk of incarceration

Infantile umbilical hernia

Symmetrical bulge under the umbilicus
More common in premature and Afro-Caribbean babies
The vast majority resolve without intervention before the age of 4-5 years
Complications are rare

Question 3 of 69
A 69-year-old man is started on tamsulosin for benign prostatic hyperplasia. Which one of the
following best describes the side-effects he may experience?
A. Urgency + insomnia
B. Dizziness + postural hypotension
C. Urinary retention + nausea
D. Urgency + erectile dysfunction
E. Erectile dysfunction + reduced libido

Benign prostatic hyperplasia
Benign prostatic hyperplasia (BPH) is a common condition seen in older men.
Risk factors



age: around 50% of 50-year-old men will have evidence of BPH and 30% will have



symptoms. Around 80% of 80-year-old men have evidence of BPH
ethnicity: black > white > Asian

BPH typically presents with lower urinary tract symptoms (LUTS), which may be categorised into:





voiding symptoms (obstructive): weak or intermittent urinary flow, straining, hesitancy,
terminal dribbling and incomplete emptying
storage symptoms (irritative) urgency, frequency, urgency incontinence and nocturia
post-micturition: dribbling
complications: urinary tract infection, retention, obstructive uropathy

Management options




watchful waiting
medication: alpha-1 antagonists, 5 alpha-reductase inhibitors. The use of combination
therapy was supported by the Medical Therapy Of Prostatic Symptoms (MTOPS) trial
surgery: transurethral resection of prostate (TURP)

Alpha-1 antagonists e.g. tamsulosin, alfuzosin


decrease smooth muscle tone (prostate and bladder)



considered first-line, improve symptoms in around 70% of men
adverse effects: dizziness, postural hypotension, dry mouth, depression



5 alpha-reductase inhibitors e.g. finasteride





block the conversion of testosterone to dihydrotestosterone (DHT), which is known to induce
BPH
unlike alpha-1 antagonists causes a reduction in prostate volume and hence may slow
disease progression. This however takes time and symptoms may not improve for 6 months.
They may also decrease PSA concentrations by up to 50%
adverse effects: erectile dysfunction, reduced libido, ejaculation problems, gynaecomastia

Question 4 of 69
Which one of the following statements regarding inguinal hernias is not correct?
A. There is no merit in differentiating between direct and indirect hernias prior to
referral
B. Patients should be referred promptly due to the risk of strangulation
C. Symptoms are typically worse following exertion
D. 95% of patients are male
E. Are the most common cause of abdominal wall hernias

Inguinal hernias rarely strangulate

Inguinal hernia
Inguinal hernias account for 75% of abdominal wall hernias. Around 95% of patients are male; men
have around a 25% lifetime risk of developing an inguinal hernia.
Features




groin lump: disappears on pressure or when the patient lies down
discomfort and ache: often worse with activity, severe pain is uncommon
strangulation is rare

Whilst traditional textbooks describe the anatomical differences between indirect (hernia through the
inguinal canal) and direct hernias (through the posterior wall of the inguinal canal) this is of no
relevance to the clinical management.
Management


the clinical consensus is currently to treat medically fit patients even if they are asymptomatic




a hernia truss may be an option for patients not fit for surgery but probably has little role in
other patients
mesh repair is associated with the lowest recurrence rate

The Department for Work and Pensions recommend that following an open repair patients return to
non-manual work after 2-3 weeks and following laparoscopic repair after 1-2 weeks
Complications



early: bruising, wound infection
late: chronic pain, recurrence

Question 5 of 69
Which one of the following is not an indication for circumcision?
A. Phimosis
B. Paraphimosis
C. Recurrent balanitis
D. Balanitis xerotica obliterans
E. Peyronie's disease

Circumcision
Circumcision has been performed in a variety of cultures for thousands of years. Today it is mainly
people of the Jewish and Islamic faith who undergo circumcision for religious/cultural reasons.
Circumcision for religious or cultural reasons is not available on the NHS.
The medical benefits of routine circumcision remain controversial although some evidence has
emerged that it:



reduces the risk of penile cancer



reduces the risk of UTI
reduces the risk of acquiring sexually transmitted infections including HIV



Medical indications for circumcision


phimosis



recurrent balanitis
balanitis xerotica obliterans
paraphimosis




It is important to exclude hypospadias prior to circumcision as the foreskin may be used in surgical
repair. Circumcision may be performed under a local or general anaesthetic.

Question 6-8 of 69
Theme: Abdominal pain
A. Myocardial infarction
B. Colorectal cancer
C. Duodenal ulcer
D. Gastric ulcer
E. Biliary colic
F. Ruptured abdominal aortic aneurysm
G. Acute pancreatitis
H. Toxic megacolon
I. Diverticulitis
J. Intestinal obstruction
For each one of the following scenarios please select the most likely diagnosis:

A 65-year-old man with a history of ischaemic heart disease presents with sudden onset central
abdominal pain radiating to his back. He is clammy and short of breath.

6.

You answered Myocardial infarction
The correct answer is Ruptured abdominal aortic aneurysm

A 34-year-old man who drinks 21 units of alcohol per week presents with episodic epigastric pain that is
relieved by eating.

7.

You answered Colorectal cancer
The correct answer is Duodenal ulcer

A 40-year-old woman with a history of Crohn's disease presents with abdominal pain and distension. She
describes constipation for the past 4 days.

8.

You answered Duodenal ulcer
The correct answer is Intestinal obstruction

Abdominal pain

The table below gives characteristic exam question features for conditions causing abdominal pain.
Unusual and 'medical' causes of abdominal pain should also be remembered:






myocardial infarction
diabetic ketoacidosis
pneumonia
acute intermittent porphyria
lead poisoning

Condition

Characteristic exam feature

Peptic ulcer disease

Duodenal ulcers: more common than gastric ulcers, epigastric pain relieved by eating
Gastric ulcers: epigastric pain worsened by eating
Features of upper gastrointestinal haemorrhage may be seen (haematemesis, melena etc)

Appendicitis

Pain initial in the central abdomen before localising to the right iliac fossa
Anorexia is common
Tachycardia, low-grade pyrexia, tenderness in RIF
Rovsing's sign: more pain in RIF than LIF when palpating LIF

Acute pancreatitis

Usually due to alcohol or gallstones
Severe epigastric pain
Vomiting is common
Examination may reveal tenderness, ileus and low-grade fever
Periumbilical discolouration (Cullen's sign) and flank discolouration (Grey-Turner's sign) is
described but rare

Biliary colic

Pain in the RUQ radiating to the back and interscapular region, may be following a fatty
meal. Slight misnomer as the pain may persist for hours
Obstructive jaundice may cause pale stools and dark urine
It is sometimes taught that patients are female, forties, fat and fair although this is obviously
a generalisation

Acute cholecystitis

History of gallstones symptoms (see above)
Continuous RUQ pain
Fever, raised inflammatory markers and white cells
Murphy's sign positive (arrest of inspiration on palpation of the RUQ)

Diverticulitis

Colicky pain typically in the LLQ
Fever, raised inflammatory markers and white cells

Abdominal aortic
aneurysm

Severe central abdominal pain radiating to the back
Presentation may be catastrophic (e.g. Sudden collapse) or sub-acute (persistent severe
central abdominal pain with developing shock)
Patients may have a history of cardiovascular disease

Intestinal obstruction

History of malignancy/previous operations
Vomiting
Not opened bowels recently
'Tinkling' bowel sounds

Question 9 of 69
Which one of the following may be used to monitor patients with colorectal cancer?
A. CA-125
B. Carcinoembryonic antigen
C. Alpha-fetoprotein
D. CA 19-9
E. CA 15-3

Carcinoembryonic antigen may be used to monitor for recurrence in patients post-operatively or to
assess response to treatment in patients with metastatic disease
Colorectal cancer: screening
Overview





most cancers develop from adenomatous polyps. Screening for colorectal cancer has been
shown to reduce mortality by 16%
the NHS now has a national screening programme offering screening every 2 years to all
men and women aged 60 to 69 years. Patients aged over 70 years may request screening
eligible patients are sent faecal occult blood (FOB) tests through the post
patients with abnormal results are offered a colonoscopy

At colonoscopy, approximately:




5 out of 10 patients will have a normal exam
4 out of 10 patients will be found to have polyps which may be removed due to their
premalignant potential
1 out of 10 patients will be found to have cancer

Question 10 of 69
A 24-year-old man presents with rectal bleeding and a 'sharp, stinging' pain on defecation. This has
been present for the past two weeks. He has a tendency towards constipation and notices that when
he wipes himself fresh blood is often on the paper. Rectal examination is limited due to pain but no
external abnormalities are seen. What is the most likely diagnosis?
A. Internal haemorrhoids
B. Anal carcinoma
C. Rectal polyp
D. Anogenital herpes
E. Anal fissure

The combination of pain and bleeding is very characteristic of anal fissures. Pain is a feature of
thrombosed external haemorrhoids but is unusual with internal haemorrhoids. Superficial anal
fissures may be difficult to see on examination.
Anal fissure
Anal fissures are longitudinal or elliptical tears of the squamous lining of the distal anal canal. If
present for less than 6 weeks they are defined as acute, and chronic if present for more than 6
weeks. Around 90% of anal fissures occur on the posterior midline
Management of an acute anal fissure (< 6 weeks)


dietary advice: high-fibre diet with high fluid intake



bulk-forming laxatives are first line - if not tolerated then lactulose should be tried
lubricants such as petroleum jelly may be tried before defecation
topical anaesthetics




-analgesia


topical steroids do not provide significant relief

Management of a chronic anal fissure (> 6 weeks)



the above techniques should be continued



topical glyceryl trinitrate (GTN) is first line treatment for a chronic anal fissure
if topical GTN is not effective after 8 weeks then secondary referral should be considered for
surgery or botulinum toxin



Question 11-13 of 69
Theme: Breast disorders
A. Lipoma
B. Paget's disease of the breast
C. Breast cancer
D. Sebaceous cysts
E. Fibroadenoma
F. Fibroadenosis
G. Duct papilloma
H. Breast abscess
I. Fat necrosis
J. Mammary duct ectasia
For each one of the following please select the most appropriate answer:

11.

A 72-year-old woman complains of 'eczema' on her left nipple. On examination the areola is
erythematous and thickened.
You answered Lipoma
The correct answer is Paget's disease of the breast

12.

A 26-year-old woman has noticed a discrete, non-tender lump which is highly mobile on examination.
You answered Paget's disease of the breast

The correct answer is Fibroadenoma

13.

A 35-year-old woman complains of 'lumpy' breasts. Her symptoms are worse in the premenstrual
period.
You answered Breast cancer
The correct answer is Fibroadenosis

Breast disorders

The table below describes some of the features seen in the most common breast disorders:

Disorder

Features

Fibroadenoma

Common in women under the age of 30 years
Often described as 'breast mice' due as they are discrete, non-tender, highly
mobile lumps

Fibroadenosis (fibrocystic disease,
benign mammary dysplasia)

Most common in middle-aged women
'Lumpy' breasts which may be painful. Symptoms may worsen prior to
menstruation

Breast cancer

Characteristically a hard, irregular lump. There may be associated nipple
inversion or skin tethering
Paget's disease of the breast - intraductal carcinoma associated with a
reddening and thickening (may resemble eczematous changes) of the
skin/areola

Mammary duct ectasia

Dilatation of the large breast ducts
Most common around the menopause
May present with a tender lump around the areola +/- a green nipple
discharge
If ruptures may cause local inflammation, sometimes referred to as 'plasma
cell mastitis'

Duct papilloma

Local areas of epithelial proliferation in large mammary ducts
Hyperplastic lesions rather than malignant or premalignant

Disorder

Features
May present with blood stained discharge

Fat necrosis

More common in obese women with large breasts
May follow trivial or unnoticed trauma
Initial inflammatory response, the lesion is typical firm and round but may
develop into a hard, irregular breast lump
Rare and may mimic breast cancer so further investigation is always
warranted

Breast abscess

More common in lactating women
Red, hot tender swelling

Lipomas and sebaceous cysts may also develop around the breast tissue.

Question 14 of 69
Which one of the following clinical features would not warrant an urgent referral (i.e. within 2 weeks)
to local colorectal services?
A. Unexplained iron-deficiency anaemia in a 50-year-old male
B. 62-year-old female with a 3 month history of rectal bleeding
C. Palpable rectal mass in a 36-year-old female
D. 48-year-old female with a 8 week history of rectal bleeding and increased stool
frequency
E. 65-year-old man with new onset constipation lasting 8 weeks

Colorectal cancer fast-track - diarrhoea is more significant than constipation

If the 65-year-old man's symptoms persist he may still require lower gastrointestinal investigations,
but he does not meet the fast-track criteria.
Colorectal cancer: referral guidelines
NICE recommend the following patients are referred urgently (i.e. within 2 weeks) to colorectal
services for investigation:







patients > 40 years old, reporting rectal bleeding with a change of bowel habit towards looser
stools and/or increased stool frequency persisting for 6 weeks or more
patients > 60 years old, with rectal bleeding persisting for 6 weeks or more without a change
in bowel habit and without anal symptoms
patients > 60 years old, with a change in bowel habit to looser stools and/or more frequent
stools persisting for 6 weeks or more without rectal bleeding
any patient presenting with a right lower abdominal mass consistent with involvement of the
large bowel
any patient with a palpable rectal mass
unexplained iron deficiency anaemia in men or non-menstruating women (Hb < 11 g/dl in
men, < 10 g/dl in women)

Question 15 of 69
Which one of the following is most associated with male infertility?
A. Sodium valproate therapy
B. Benign prostatic hyperplasia
C. Varicoceles
D. Epididymal cysts
E. Hydroceles

Varicoceles may be associated with infertility

Scrotal problems
Epididymal cysts
Epididymal cysts are the most common cause of scrotal swellings seen in primary care.
Features



separate from the body of the testicle
found posterior to the testicle

Associated conditions




polycystic kidney disease
cystic fibrosis
von Hippel-Lindau syndrome

Diagnosis may be confirmed by ultrasound.
Management is usually supportive but surgical removal or sclerotherapy may be attempted for larger
or symptomatic cysts.
Hydrocele
A hydrocele describes the accumulation of fluid within the tunica vaginalis. They can be divided into
communicating and non-communicating:




communicating: caused by patency of the processus vaginalis allowing peritoneal fluid to
drain down into the scrotum. Communicating hydroceles are common in newborn males
(clinically apparent in 5-10%) and usually resolve within the first few months of life
non-communicating: caused by excessive fluid production within the tunica vaginalis

Hydroceles may develop secondary to:




epididymo-orchitis
testicular torsion
testicular tumours

Features


soft, non-tender swelling of the hemi-scrotum. Usually anterior to and below the testicle



the swelling is confined to the scrotum, you can get 'above' the mass on examination
transilluminates with a pen torch
the testis may be difficult to palpate if the hydrocele is large




Diagnosis may be clinical but ultrasound is required if there is any doubt about the diagnosis or if the
underlying testis cannot be palpated.
Management



infantile hydroceles are generally repaired if they do not resolve spontaneously by the age of
1-2 years
in adults a conservative approach may be taken depending on the severity of the
presentation. Further investigation (e.g. ultrasound) is usually warranted however to exclude
any underlying cause such as a tumour

Varicocele
A varicocele is an abnormal enlargement of the testicular veins. They are usually asymptomatic but
may be important as they are associated with infertility.
Varicoceles are much more common on the left side (> 80%). Features:



classically described as a 'bag of worms'
subfertility

Diagnosis


ultrasound with Doppler studies

Management



usually conservative
occasionally surgery is required if the patient is troubled by pain. There is ongoing debate
regarding the effectiveness of surgery to treat infertility

Question 16 of 69
A 62-year-old man presents with nocturia, hesitancy and terminal dribbling. Prostate examination
reveals a moderately enlarged prostate with no irregular features and a well defined median sulcus.
Blood tests show:

PSA 1.3 ng/ml
What is the most appropriate management?
A. Alpha-1 antagonist
B. 5 alpha-reductase inhibitor
C. Non-urgent referral for transurethral resection of prostate
D. Empirical treatment with ciprofloxacin for 2 weeks
E. Urgent referral to urology

Alpha-1 antagonists are first-line in patients with benign prostatic hyperplasia
Benign prostatic hyperplasia
Benign prostatic hyperplasia (BPH) is a common condition seen in older men.
Risk factors


age: around 50% of 50-year-old men will have evidence of BPH and 30% will have
symptoms. Around 80% of 80-year-old men have evidence of BPH



ethnicity: black > white > Asian

BPH typically presents with lower urinary tract symptoms (LUTS), which may be categorised into:


voiding symptoms (obstructive): weak or intermittent urinary flow, straining, hesitancy,
terminal dribbling and incomplete emptying



storage symptoms (irritative) urgency, frequency, urgency incontinence and nocturia
post-micturition: dribbling
complications: urinary tract infection, retention, obstructive uropathy




Management options




watchful waiting
medication: alpha-1 antagonists, 5 alpha-reductase inhibitors. The use of combination
therapy was supported by the Medical Therapy Of Prostatic Symptoms (MTOPS) trial
surgery: transurethral resection of prostate (TURP)

Alpha-1 antagonists e.g. tamsulosin, alfuzosin




decrease smooth muscle tone (prostate and bladder)
considered first-line, improve symptoms in around 70% of men
adverse effects: dizziness, postural hypotension, dry mouth, depression

5 alpha-reductase inhibitors e.g. finasteride





block the conversion of testosterone to dihydrotestosterone (DHT), which is known to induce
BPH
unlike alpha-1 antagonists causes a reduction in prostate volume and hence may slow
disease progression. This however takes time and symptoms may not improve for 6 months.
They may also decrease PSA concentrations by up to 50%
adverse effects: erectile dysfunction, reduced libido, ejaculation problems, gynaecomastia

Question 17 of 69
Which one of the following types of suture is absorbable?
A. Ethilon
B. Vicryl
C. Novafil
D. Prolene
E. Silk

Minor surgery
Local anaesthetic (LA)
Lidocaine is the most widely used LA. It has a rapid onset of action and anaesthesia lasts for around
1 hour.


the maximum safe dose is 3mg/kg. The BNF states 200mg (or 500mg if given in solutions
containing adrenaline), which equates to 3mg/kg for a 66kg patient. This is the equivalent of
20ml of 1% solution or 10ml of 2% solution



lidocaine is available pre-mixed with adrenaline. This increases the duration of action of
lidocaine and reduces blood loss secondary to vasoconstriction. It must never be used near
extremities due to the risk of ischaemia

Suture material

Non-absorbable Absorbable
Silk
Novafil
Prolene
Ethilon

Vicryl
Dexon
PDS

Non-absorbable sutures are normally removed after 7-14 days, depending on the location.

Absorbable sutures normally disappear after 7-10 days. Removal times for non-absorbable sutures
are shown below:

Area

Removal time (days)

Face

3-5

Scalp, limbs, chest

7 - 10

Hand, foot, back

10 - 14

Question 18 of 69
A 62-year-old man with no significant past medical history presents with a right sided groin lump
which he noticed whilst having a shower. It has been present for 2 weeks and disappears when he
lies down. It never causes him any discomfort and there are no other gastrointestinal symptoms of
note. Examination reveals an small reducible swelling in the right groin. What is the most appropriate
management?
A. Refer for fitting of a truss
B. Refer to vascular surgeon
C. Routine referral for surgical repair
D. Advise no action as it will probably improve with time
E. Fast-track referral to colorectal service
Next question
This patient has an asymptomatic inguinal hernia. Studies looking at conservative management tend
to find that many patients become symptomatic and eventually have surgery anyway. As this patient
is medically fit most clinicians would refer for surgical repair.
Inguinal hernias do not resolve spontaneously.

A number of PCTs have begun to put asymptomatic inguinal hernias on the 'low clinical priority' list.
Whilst this may be reasonable for older patients who are 'not bothered' by their condition it is
debatable how feasible such a blanket policy is for all patients.
Inguinal hernia
Inguinal hernias account for 75% of abdominal wall hernias. Around 95% of patients are male; men
have around a 25% lifetime risk of developing an inguinal hernia.
Features




groin lump: disappears on pressure or when the patient lies down
discomfort and ache: often worse with activity, severe pain is uncommon
strangulation is rare

Whilst traditional textbooks describe the anatomical differences between indirect (hernia through the
inguinal canal) and direct hernias (through the posterior wall of the inguinal canal) this is of no
relevance to the clinical management.
Management




the clinical consensus is currently to treat medically fit patients even if they are asymptomatic
a hernia truss may be an option for patients not fit for surgery but probably has little role in
other patients
mesh repair is associated with the lowest recurrence rate

The Department for Work and Pensions recommend that following an open repair patients return to
non-manual work after 2-3 weeks and following laparoscopic repair after 1-2 weeks
Complications



early: bruising, wound infection
late: chronic pain, recurrence

Question 19 of 69
A 30-year-old man presents with a painless lump in his right testicle. Which one of the following is
most strongly associated with testicular cancer?
A. Increasing age
B. Smoking
C. Infertility
D. High maternal age
E. High paternal age

Testicular cancer
Testicular cancer is the most common malignancy in men aged 20-30 years. Around 95% of cases
of testicular cancer are germ-cell tumours. Germ cell tumours may essentially be divided into:



seminomas
teratomas

Other type of germ cell tumours include yolk sac tumours. Non-germ cell tumours include Leydig cell
tumours and sarcomas.
The peak incidence for teratomas is 25 years and seminomas is 35 years. Risk factors include:






cryptorchidism
infertility
family history
Klinefelter's syndrome
mumps orchitis

Features



a painless lump is the most common presenting symptom



pain may also be present in a minority of men
other possible features include hydrocele, gynaecomastia



Diagnosis


ultrasound is first-line

Management



orchidectomy
chemotherapy and radiotherapy may be given depending on staging

Prognosis is generally excellent



5 year survival for seminomas is around 95% if Stage I
5 year survival for teratomas is around 85% if Stage I

Question 20 of 69
A 37-year-old man with a history of internal haemorrhoids presents as his symptoms have recently
flared. He now describes piles which he has to manually reduce following defecation. What grade of
haemorrhoids does he have?
A. Grading system does not apply to internal haemorrhoids
B. Grade I
C. Grade II
D. Grade III
E. Grade IV

Haemorrhoids
Haemorrhoidal tissue is part of the normal anatomy which contributes to anal continence. These
mucosal vascular cushions are found in the left lateral, right posterior and right anterior portions of
the anal canal (3 o'clock, 7'o'clock and 11 o'clock respectively). Haemorrhoids are said to exist when
they become enlarged, congested and symptomatic
Clinical features





painless rectal bleeding is the most common symptom
pruritus
pain: usually not significant unless piles are thrombosed
soiling may occur with third or forth degree piles

Types of haemorrhoids
External



originate below the dentate line
prone to thrombosis, may be painful

Internal



originate above the dentate line
do not generally cause pain

Grading of internal haemorrhoids

Grade I

Do not prolapse out of the anal canal

Grade II

Prolapse on defecation but reduce spontaneously

Grade III

Can be manually reduced

Grade IV

Cannot be reduced

Management






soften stools: increase dietary fibre and fluid intake
topical local anaesthetics and steroids may be used to help symptoms
outpatient treatments: rubber band ligation is superior to injection sclerotherapy
surgery is reserved for large symptomatic haemorrhoids which do not respond to outpatient
treatments
newer treatments: Doppler guided haemorrhoidal artery ligation, stapled haemorrhoidopexy

Acutely thrombosed external haemorrhoids




typically present with significant pain
examination reveals a purplish, oedematous, tender subcutaneous perianal mass
if patient presents within 72 hours then referral should be considered for excision. Otherwise
patients can usually be managed with stool softeners, ice packs and analgesia. Symptoms
usually settle within 10 days

Question 21 of 69
Which one of the following statements regarding hydroceles is correct?
A. Communicating hydroceles are found in more than 3% of newborn males
B. The vast majority occur on the right hand side
C. In younger children are often secondary to a varicocele
D. With hydroceles you usually cannot get above the swelling on examination
E. Are associated with infertility

Scrotal problems

Epididymal cysts
Epididymal cysts are the most common cause of scrotal swellings seen in primary care.
Features



separate from the body of the testicle
found posterior to the testicle

Associated conditions




polycystic kidney disease
cystic fibrosis
von Hippel-Lindau syndrome

Diagnosis may be confirmed by ultrasound.
Management is usually supportive but surgical removal or sclerotherapy may be attempted for larger
or symptomatic cysts.
Hydrocele
A hydrocele describes the accumulation of fluid within the tunica vaginalis. They can be divided into
communicating and non-communicating:




communicating: caused by patency of the processus vaginalis allowing peritoneal fluid to
drain down into the scrotum. Communicating hydroceles are common in newborn males
(clinically apparent in 5-10%) and usually resolve within the first few months of life
non-communicating: caused by excessive fluid production within the tunica vaginalis

Hydroceles may develop secondary to:




epididymo-orchitis
testicular torsion
testicular tumours

Features



soft, non-tender swelling of the hemi-scrotum. Usually anterior to and below the testicle



the swelling is confined to the scrotum, you can get 'above' the mass on examination
transilluminates with a pen torch
the testis may be difficult to palpate if the hydrocele is large




Diagnosis may be clinical but ultrasound is required if there is any doubt about the diagnosis or if the
underlying testis cannot be palpated.
Management



infantile hydroceles are generally repaired if they do not resolve spontaneously by the age of
1-2 years
in adults a conservative approach may be taken depending on the severity of the
presentation. Further investigation (e.g. ultrasound) is usually warranted however to exclude
any underlying cause such as a tumour

Varicocele
A varicocele is an abnormal enlargement of the testicular veins. They are usually asymptomatic but
may be important as they are associated with infertility.
Varicoceles are much more common on the left side (> 80%). Features:



classically described as a 'bag of worms'
subfertility

Diagnosis


ultrasound with Doppler studies

Management



usually conservative
occasionally surgery is required if the patient is troubled by pain. There is ongoing debate
regarding the effectiveness of surgery to treat infertility

Question 22 of 69
A 53-year-old man who has no past history of note requests a PSA test. One of his father's friends
has recently been diagnosed with prostate cancer. What is the most appropriate action?
A. Perform a digital rectal examination and refer him to urology so he can be
counselled regarding the PSA test
B. Tell him that you can appreciate his concern but reassure that at his age he is at
very low risk
C. Advise him that a national screening programme was started in 2009 and he
will be called at the age of 60 years for a test
D. Give him a patient information leaflet with details of the PSA test and allow
him to make the choice
E. Offer to perform a digital rectal examination but advise him that the PSA test
is not recommended in younger asymptomatic men

Prostate cancer: PSA testing
Prostate specific antigen (PSA) is a serine protease enzyme produced by normal and malignant
prostate epithelial cells. It has become an important tumour marker but much controversy still exists
regarding its usefulness as a screening tool.
The NHS Prostate Cancer Risk Management Programme (PCRMP) has published updated
guidelines in 2009 on how to handle requests for PSA testing in asymptomatic men. A recent
European trial (ERSPC) showed a statistically significant reduction in the rate of death prostate
cancer by 20% in men aged 55 to 69 years but this was associated with a high risk of over-diagnosis
and over-treatment. Having reviewed this and other data the National Screening Committee have
decided not to introduce a prostate cancer screening programme yet but rather allow men to make
an informed choice.

Age-adjusted upper limits for PSA were recommended by the PCRMP:

Age

PSA level (ng/ml)

50-59 years

3.0

60-69 years

4.0

> 70 years

5.0

PSA levels may also be raised by*:







benign prostatic hyperplasia (BPH)
prostatitis and urinary tract infection (NICE recommend to postpone the PSA test for at least
1 month after treatment)
ejaculation (ideally not in the previous 48 hours)
vigorous exercise (ideally not in the previous 48 hours)
urinary retention
instrumentation of the urinary tract

Poor specificity and sensitivity




around 33% of men with a PSA of 4-10 ng/ml will be found to have prostate cancer. With a
PSA of 10-20 ng/ml this rises to 60% of men
around 20% with prostate cancer have a normal PSA
various methods are used to try and add greater meaning to a PSA level including ageadjusted upper limits and monitoring change in PSA level with time (PSA velocity or PSA
doubling time)

*whether digital rectal examination actually causes a rise in PSA levels is a matter of debate

Question 23 of 69
Which one of the following scenarios would not warrant an urgent referral to the local breast service
according to NICE guidelines?
A. 25-year-old female with a 6 week history of a new breast lump. Family history
of breast cancer. Benign in nature on examination
B. 34-year-old female with a 6 week history of a new breast lump. Benign in
nature on examination
C. 55-year-old female with new breast lump. Benign in nature on examination
D. 28-year-old female with a 8 week history of a new breast lump. Benign in
nature on examination
E. 29-year-old female with a unilateral bloody nipple discharge

NICE guidelines suggest a cut-off age of 30 years for a breast lump without features suggestive of
cancer, but which persists after the next period. As this 28-year-old is below this cut-off she should
be referred non-urgently to the local breast services. The 25-year-old should be fast-tracked as she
has a family history of breast cancer.
Breast cancer: referral
NICE published referral guidelines for suspected breast cancer in 2005
Urgent referrals (i.e. within 2 weeks)







any breast lump with features suggestive of cancer (hard, tethered etc)
any breast lump in a post-menopausal woman, regardless of features suggestive of cancer
any breast lump in a women more than 30 years old without features suggestive of cancer
but which persists after her next period
if there is past history of breast cancer any breast lump should warrant urgent referral
spontaneous unilateral bloody nipple discharge
unilateral eczematous skin or nipple change that does not respond to topical treatment, or
with nipple distortion of recent onset

Non-urgent referrals



women < 30 years old who present with a breast lump with no features suggestive of cancer,
no relevant family history and no change in the size of the lump

Question 24-26 of 69
Theme: Abdominal swelling
A. Irritable bowel syndrome
B. Endometrial cancer
C. Ovarian cancer
D. Pregnancy
E. Intestinal obstruction
F. Urinary retention
G. Ascites
H. Gastric cancer
I. Colorectal cancer
J. Bladder cancer
For each one of the following scenarios select the most likely diagnosis:

24.

62-year-old woman with a 3 month history of urinary symptoms, early satiety and a raised CA125
Ovarian cancer
Ovarian cancer tends to present late due to non-specific symptoms

25.

26-year-old female with a history of constipation, episodic abdominal pain and bloating.
Irritable bowel syndrome
These are classic symptoms of irritable bowel syndrome

26.

72-year-old woman with a history of congestive cardiac failure. She reports having a poor appetite and
feeling bloated. She is admitted frequently to hospital with left ventricular failure due to poor
compliance with medication
Ascites

Patients with poorly controlled heart failure may develop 'cardiac cachexia', partly due to gut oedema

Abdominal swelling

The table below gives characteristic exam question features for conditions causing abdominal
swelling

Condition

Characteristic exam feature

Pregnancy

Young female
Amenorrhoea

Intestinal obstruction

History of malignancy/previous operations
Vomiting
Not opened bowels recently
'Tinkling' bowel sounds

Ascites

History of alcohol excess, cardiac failure

Urinary retention

History of prostate problems
Dullness to percussion around suprapubic area

Ovarian cancer

Older female
Pelvic pain
Urinary symptoms e.g. urgency
Raised CA125
Early satiety, bloating

Question 27 of 69
A 45-year woman who you have treated for obesity comes for review. Despite ongoing lifestyle
interventions and trials of orlistat and sibutramine she has failed to lose a significant amount of
weight. She is currently taking ramipril for hypertension but a recent fasting glucose was normal. For
this patient, what is the cut-off body mass index (BMI) that would trigger a referral for consideration
of bariatric surgery?
A. BMI > 35 kg/m^2
B. BMI > 40 kg/m^2
C. BMI > 30 kg/m^2
D. BMI > 38 kg/m^2
E. BMI > 45 kg/m^2

Obesity - NICE bariatric referral cut-offs



with risk factors (T2DM, BP etc): > 35 kg/m^2
no risk factors: > 40 kg/m^2

Obesity: bariatric surgery
The use of bariatric surgery in the management of obesity has developed significantly over the past
decade. It is now recognised that for many obese patients who fail to lose weight with lifestyle and
drug interventions the risks and expense of long-term obesity outweigh those of surgery.
NICE guidelines on bariatric surgery for adults
Consider surgery for people with severe obesity if:








they have a BMI of 40 kg/m^2 or more, or between 35 kg/m^2 and 40 kg/m^2 and other
significant disease (for example, type 2 diabetes mellitus, hypertension) that could be
improved if they lost weight
all appropriate non-surgical measures have failed to achieve or maintain adequate clinically
beneficial weight loss for at least 6 months
they are receiving or will receive intensive specialist management
they are generally fit for anaesthesia and surgery
they commit to the need for long-term follow-up

Consider surgery as a first-line option for adults with a BMI of more than 50 kg/m2 in whom surgical
intervention is considered appropriate; consider orlistat before surgery if the waiting time is long
Types of bariatric surgery:




primarily restrictive: laparoscopic-adjustable gastric banding (LAGB) or sleeve gastrectomy
primarily malabsorptive: classic biliopancreatic diversion (BPD) has now largely been
replaced by biliopancreatic diversion with duodenal switch
mixed: Roux-en-Y gastric bypass surgery

Which operation?





LAGB produces less weight loss than malabsorptive or mixed procedures but as it has fewer
complications it is normally the first-line intervention in patients with a BMI of 30-39kg/m^2
patients with a BMI > 40 kg/m^2 may be considered for a gastric bypass or sleeve
gastrectomy. The latter may be done as a sole procedure or as an initial procedure prior to
bypass
primarily malabsorptive procedures are usually reserved for very obese patients (e.g. BMI >
60 kg/m^2)

Question 28 of 69
Which one of the following ethnic groups have an increased incidence of prostate cancer?
A. Afro-Caribbean
B. Ashkenazi Jews
C. Chinese
D. Indian subcontinent
E. White

Prostate cancer - more common in the Afro-Caribbean population

Prostate cancer: features
Prostate cancer is now the most common cancer in adult males in the UK and is the second most
common cause of death due to cancer in men after lung cancer.
Risk factors





increasing age
obesity
Afro-Caribbean ethnicity
family history: around 5-10% of cases have a strong family history

Localised prostate cancer is often asymptomatic. This is partly because cancers tend to develop in
the periphery of the prostate and hence don't cause obstructive symptoms early on. Possible
features include:




bladder outlet obstruction: hesitancy, urinary retention
haematuria, haematospermia
pain: back, perineal or testicular



digital rectal examination: asymmetrical, hard, nodular enlargement with loss of median
sulcus

Question 29 of 69
A 24-year-old man presents due to severe pain when defecating for the past 2 weeks. He has
occasionally noted some blood on the toilet paper when wiping himself. On examination a tear is
seen on the posterior midline of the anal verge. Which one of the following should not be
recommended as a treatment option?
A. Bulk-forming laxatives
B. Application of lubricant prior to defecation
C. Topical steroids
D. Dietary advice
E. Paracetamol

Topical steroids have been shown in studies to be of little benefit in treating anal fissures
Anal fissure
Anal fissures are longitudinal or elliptical tears of the squamous lining of the distal anal canal. If
present for less than 6 weeks they are defined as acute, and chronic if present for more than 6
weeks. Around 90% of anal fissures occur on the posterior midline
Management of an acute anal fissure (< 6 weeks)





dietary advice: high-fibre diet with high fluid intake
bulk-forming laxatives are first line - if not tolerated then lactulose should be tried
lubricants such as petroleum jelly may be tried before defecation
topical anaesthetics

-analgesia



topical steroids do not provide significant relief

Management of a chronic anal fissure (> 6 weeks)




the above techniques should be continued
topical glyceryl trinitrate (GTN) is first line treatment for a chronic anal fissure
if topical GTN is not effective after 8 weeks then secondary referral should be considered for
surgery or botulinum toxin

Question 30-32 of 69
Theme: Abdominal pain
A. Alcoholic hepatitis
B. Acute cholecystitis
C. Duodenal ulcer
D. Gastric ulcer
E. Biliary colic
F. Ruptured abdominal acute aneurysm
G. Acute pancreatitis
H. Gastroenteritis
I. Diverticulitis
J. Intestinal obstruction
For each one of the following scenarios please select the most likely diagnosis:

30.

A 49-year-old woman presents with pain in the right upper quadrant. This has been occurring for the
past 3 months and is often precipitated by a heavy meal. When the pain comes it is typically lasts
around 1-2 hours. Clinical examination is unremarkable other than mild tenderness in the right upper
quadrant.

The correct answer is Biliary colic

A 37-year-old attends surgery due to a one day history of severe central abdominal pain radiating
through to the back. He has vomited several times and is guarding on examination. Parotitis and spider
naevi are also noted.

31.

The correct answer is Acute pancreatitis
Parotitis and spider naevi suggest excessive alcohol intake which is one of the most common causes of
acute pancreatitis.

A 72-year-old woman who takes regular laxatives comes to surgery. Over the past two days she has
developed progressively worse pain in the left lower quadrant. On examination she has a low-grade
pyrexia and is tender on the left side of the abdomen

32.

The correct answer is Diverticulitis

Abdominal pain

The table below gives characteristic exam question features for conditions causing abdominal pain.
Unusual and 'medical' causes of abdominal pain should also be remembered:






myocardial infarction
diabetic ketoacidosis
pneumonia
acute intermittent porphyria
lead poisoning

Condition
Peptic ulcer disease

Characteristic exam feature
Duodenal ulcers: more common than gastric ulcers, epigastric pain relieved by eating
Gastric ulcers: epigastric pain worsened by eating

Condition

Characteristic exam feature
Features of upper gastrointestinal haemorrhage may be seen (haematemesis, melena etc)

Appendicitis

Pain initial in the central abdomen before localising to the right iliac fossa
Anorexia is common
Tachycardia, low-grade pyrexia, tenderness in RIF
Rovsing's sign: more pain in RIF than LIF when palpating LIF

Acute pancreatitis

Usually due to alcohol or gallstones
Severe epigastric pain
Vomiting is common
Examination may reveal tenderness, ileus and low-grade fever
Periumbilical discolouration (Cullen's sign) and flank discolouration (Grey-Turner's sign) is
described but rare

Biliary colic

Pain in the RUQ radiating to the back and interscapular region, may be following a fatty
meal. Slight misnomer as the pain may persist for hours
Obstructive jaundice may cause pale stools and dark urine
It is sometimes taught that patients are female, forties, fat and fair although this is obviously
a generalisation

Acute cholecystitis

History of gallstones symptoms (see above)
Continuous RUQ pain
Fever, raised inflammatory markers and white cells
Murphy's sign positive (arrest of inspiration on palpation of the RUQ)

Diverticulitis

Colicky pain typically in the LLQ
Fever, raised inflammatory markers and white cells

Abdominal aortic
aneurysm

Severe central abdominal pain radiating to the back
Presentation may be catastrophic (e.g. Sudden collapse) or sub-acute (persistent severe
central abdominal pain with developing shock)
Patients may have a history of cardiovascular disease

Intestinal obstruction

History of malignancy/previous operations
Vomiting
Not opened bowels recently
'Tinkling' bowel sounds

Question 33 of 69
A patient is started on finasteride for the treatment of benign prostatic hyperplasia. How long should
the patient be told that treatment may take to be effective?
A. Within 8 hours of taking the tablet
B. Within 3 days
C. Up to 7 days
D. Up to 4 weeks
E. Up to 6 months

Finasteride treatment of BPH may take 6 months before results are seen

Benign prostatic hyperplasia
Benign prostatic hyperplasia (BPH) is a common condition seen in older men.
Risk factors



age: around 50% of 50-year-old men will have evidence of BPH and 30% will have
symptoms. Around 80% of 80-year-old men have evidence of BPH
ethnicity: black > white > Asian

BPH typically presents with lower urinary tract symptoms (LUTS), which may be categorised into:





voiding symptoms (obstructive): weak or intermittent urinary flow, straining, hesitancy,
terminal dribbling and incomplete emptying
storage symptoms (irritative) urgency, frequency, urgency incontinence and nocturia
post-micturition: dribbling
complications: urinary tract infection, retention, obstructive uropathy

Management options




watchful waiting
medication: alpha-1 antagonists, 5 alpha-reductase inhibitors. The use of combination
therapy was supported by the Medical Therapy Of Prostatic Symptoms (MTOPS) trial
surgery: transurethral resection of prostate (TURP)

Alpha-1 antagonists e.g. tamsulosin, alfuzosin


decrease smooth muscle tone (prostate and bladder)



considered first-line, improve symptoms in around 70% of men
adverse effects: dizziness, postural hypotension, dry mouth, depression



5 alpha-reductase inhibitors e.g. finasteride





block the conversion of testosterone to dihydrotestosterone (DHT), which is known to induce
BPH
unlike alpha-1 antagonists causes a reduction in prostate volume and hence may slow
disease progression. This however takes time and symptoms may not improve for 6 months.
They may also decrease PSA concentrations by up to 50%
adverse effects: erectile dysfunction, reduced libido, ejaculation problems, gynaecomastia

Question 34 of 69
Which one of the following scenarios is the most common presentation of testicular cancer?
A. Painful testicular lump in a 56-year-old man
B. Painless testicular lump in a 27-year-old man
C. Painless testicular lump in a 43-year-old man
D. Painful testicular lump in a 25-year-old man
E. Painful testicular lump associated with dysuria in a 38-year-old man

Testicular cancer
Testicular cancer is the most common malignancy in men aged 20-30 years. Around 95% of cases
of testicular cancer are germ-cell tumours. Germ cell tumours may essentially be divided into:


seminomas



teratomas
Other type of germ cell tumours include yolk sac tumours. Non-germ cell tumours include
Leydig cell tumours and sarcomas.
The peak incidence for teratomas is 25 years and seminomas is 35 years. Risk factors
include:







cryptorchidism
infertility
family history
Klinefelter's syndrome
mumps orchitis

Features




a painless lump is the most common presenting symptom
pain may also be present in a minority of men
other possible features include hydrocele, gynaecomastia

Diagnosis


ultrasound is first-line

Management






orchidectomy
chemotherapy and radiotherapy may be given depending on staging
Prognosis is generally excellent
5 year survival for seminomas is around 95% if Stage I
5 year survival for teratomas is around 85% if Stage I

Question 35 of 69
A 25-year-old female presents to surgery with a 2 week history of painless rectal bleeding.
Inspection of perineum and rectal examination is unremarkable. Proctoscopy reveals haemorrhoidal
cushions at the left lateral and right anterior position. What is the most important component of
management?
A. Sitz baths
B. Topical nitrate
C. Fibre supplementation
D. Improving anal hygiene
E. Application of lubricant prior to defecation

Fibre supplementation has been shown to be as effective as injection sclerotherapy in some studies
Haemorrhoids
Haemorrhoidal tissue is part of the normal anatomy which contributes to anal continence. These
mucosal vascular cushions are found in the left lateral, right posterior and right anterior portions of
the anal canal (3 o'clock, 7'o'clock and 11 o'clock respectively). Haemorrhoids are said to exist when
they become enlarged, congested and symptomatic
Clinical features





painless rectal bleeding is the most common symptom
pruritus
pain: usually not significant unless piles are thrombosed
soiling may occur with third or forth degree piles

Types of haemorrhoids
External



originate below the dentate line
prone to thrombosis, may be painful

Internal



originate above the dentate line
do not generally cause pain

Grading of internal haemorrhoids

Grade I

Do not prolapse out of the anal canal

Grade II

Prolapse on defecation but reduce spontaneously

Grade III

Can be manually reduced

Grade IV

Cannot be reduced

Management






soften stools: increase dietary fibre and fluid intake
topical local anaesthetics and steroids may be used to help symptoms
outpatient treatments: rubber band ligation is superior to injection sclerotherapy
surgery is reserved for large symptomatic haemorrhoids which do not respond to outpatient
treatments
newer treatments: Doppler guided haemorrhoidal artery ligation, stapled haemorrhoidopexy

Acutely thrombosed external haemorrhoids




typically present with significant pain
examination reveals a purplish, oedematous, tender subcutaneous perianal mass
if patient presents within 72 hours then referral should be considered for excision. Otherwise
patients can usually be managed with stool softeners, ice packs and analgesia. Symptoms
usually settle within 10 days

Question 36-38 of 69

Theme: Scrotal problems
A. Varicocele
B. Testicular cancer
C. Epididymo-orchitis
D. Epididymal cyst
E. Inguinal hernia
F. Hydrocele
G. Femoral hernia
H. Hydatid of Morgagni
I. Fournier's gangrene
J. Cardiac failure
For each of the following scenarios please select the most likely diagnosis:

36.

A 31-year-old man presents as he and his partner have been having problems conceiving. On
examination there is a diffuse lumpy swelling on the left side of his scrotum. This is not painful and the
testicle, which can be felt separately, is normal.
Varicocele

37.

A 44-year-old man notices a pea-sized lump on his right testicle. On examination a discrete soft mass
can be felt posterior to the right testicle.

The correct answer is Epididymal cyst

38.

A 75-year-old man presents with a swelling in his right scrotum. On examination a large, non-tender
swelling is found in the scrotum. You cannot palpate above the swelling during the examination.

The correct answer is Inguinal hernia
A hydrocele is less likely as you cannot 'get above' the swelling on examination.

Scrotal problems

Epididymal cysts
Epididymal cysts are the most common cause of scrotal swellings seen in primary care.
Features



separate from the body of the testicle
found posterior to the testicle

Associated conditions




polycystic kidney disease
cystic fibrosis
von Hippel-Lindau syndrome

Diagnosis may be confirmed by ultrasound.
Management is usually supportive but surgical removal or sclerotherapy may be attempted for larger
or symptomatic cysts.
Hydrocele
A hydrocele describes the accumulation of fluid within the tunica vaginalis. They can be divided into
communicating and non-communicating:




communicating: caused by patency of the processus vaginalis allowing peritoneal fluid to
drain down into the scrotum. Communicating hydroceles are common in newborn males
(clinically apparent in 5-10%) and usually resolve within the first few months of life
non-communicating: caused by excessive fluid production within the tunica vaginalis

Hydroceles may develop secondary to:


epididymo-orchitis



testicular torsion



testicular tumours

Features





soft, non-tender swelling of the hemi-scrotum. Usually anterior to and below the testicle
the swelling is confined to the scrotum, you can get 'above' the mass on examination
transilluminates with a pen torch
the testis may be difficult to palpate if the hydrocele is large

Diagnosis may be clinical but ultrasound is required if there is any doubt about the diagnosis or if the
underlying testis cannot be palpated.
Management



infantile hydroceles are generally repaired if they do not resolve spontaneously by the age of
1-2 years
in adults a conservative approach may be taken depending on the severity of the
presentation. Further investigation (e.g. ultrasound) is usually warranted however to exclude
any underlying cause such as a tumour

Varicocele
A varicocele is an abnormal enlargement of the testicular veins. They are usually asymptomatic but
may be important as they are associated with infertility.
Varicoceles are much more common on the left side (> 80%). Features:



classically described as a 'bag of worms'
subfertility

Diagnosis


ultrasound with Doppler studies

Management


usually conservative



occasionally surgery is required if the patient is troubled by pain. There is ongoing debate
regarding the effectiveness of surgery to treat infertility

Question 39 of 69
A 55-year-old accountant presents to surgery requesting a sick note following an open repair of an
inguinal hernia. According to Department of Work and Pensions advice, when should he be able to
return to work?
A. After 5 days
B. After 7 days
C. After 1 - 2 weeks
D. After 2 - 3 weeks
E. After 3 - 4 weeks

Inguinal hernia repair: back to work after 2-3 weeks if open, 1-2 weeks if laparoscopic

Inguinal hernia
Inguinal hernias account for 75% of abdominal wall hernias. Around 95% of patients are male; men
have around a 25% lifetime risk of developing an inguinal hernia.
Features




groin lump: disappears on pressure or when the patient lies down
discomfort and ache: often worse with activity, severe pain is uncommon
strangulation is rare

Whilst traditional textbooks describe the anatomical differences between indirect (hernia through the

inguinal canal) and direct hernias (through the posterior wall of the inguinal canal) this is of no
relevance to the clinical management.
Management


the clinical consensus is currently to treat medically fit patients even if they are asymptomatic



a hernia truss may be an option for patients not fit for surgery but probably has little role in
other patients
mesh repair is associated with the lowest recurrence rate



The Department for Work and Pensions recommend that following an open repair patients return to
non-manual work after 2-3 weeks and following laparoscopic repair after 1-2 weeks
Complications



early: bruising, wound infection
late: chronic pain, recurrence

Question 40 of 69
Which one of the following statements regarding varicoceles is correct?
A. Over 80% occur on the left side
B. All patients should be offered surgery to prevent infertility
C. Around 5% of patients have an underlying testicular cancer
D. They are more common in pre-pubertal males
E. Having a varicocele is a risk factor for deep vein thrombosis

Scrotal problems

Epididymal cysts
Epididymal cysts are the most common cause of scrotal swellings seen in primary care.
Features



separate from the body of the testicle
found posterior to the testicle

Associated conditions




polycystic kidney disease
cystic fibrosis
von Hippel-Lindau syndrome

Diagnosis may be confirmed by ultrasound.
Management is usually supportive but surgical removal or sclerotherapy may be attempted for larger
or symptomatic cysts.
Hydrocele
A hydrocele describes the accumulation of fluid within the tunica vaginalis. They can be divided into
communicating and non-communicating:




communicating: caused by patency of the processus vaginalis allowing peritoneal fluid to
drain down into the scrotum. Communicating hydroceles are common in newborn males
(clinically apparent in 5-10%) and usually resolve within the first few months of life
non-communicating: caused by excessive fluid production within the tunica vaginalis

Hydroceles may develop secondary to:




epididymo-orchitis
testicular torsion
testicular tumours

Features



soft, non-tender swelling of the hemi-scrotum. Usually anterior to and below the testicle



the swelling is confined to the scrotum, you can get 'above' the mass on examination
transilluminates with a pen torch
the testis may be difficult to palpate if the hydrocele is large




Diagnosis may be clinical but ultrasound is required if there is any doubt about the diagnosis or if the
underlying testis cannot be palpated.
Management



infantile hydroceles are generally repaired if they do not resolve spontaneously by the age of
1-2 years
in adults a conservative approach may be taken depending on the severity of the
presentation. Further investigation (e.g. ultrasound) is usually warranted however to exclude
any underlying cause such as a tumour

Varicocele
A varicocele is an abnormal enlargement of the testicular veins. They are usually asymptomatic but
may be important as they are associated with infertility.
Varicoceles are much more common on the left side (> 80%). Features:



classically described as a 'bag of worms'
subfertility

Diagnosis


ultrasound with Doppler studies

Management



usually conservative
occasionally surgery is required if the patient is troubled by pain. There is ongoing debate
regarding the effectiveness of surgery to treat infertility

Question 41 of 69
A 60-year-old man presents with lower urinary tract symptoms and is offered a PSA test. According
to NHS guidelines, which one of the following could interfere with the PSA level?
A. Vigorous exercise in the past 48 hours
B. Poorly controlled diabetes mellitus
C. Smoking in the past 48 hours
D. Current constipation
E. Drinking more than 4 units of alcohol in the past 48 hours

Prostate cancer: PSA testing
Prostate specific antigen (PSA) is a serine protease enzyme produced by normal and malignant
prostate epithelial cells. It has become an important tumour marker but much controversy still exists
regarding its usefulness as a screening tool.
The NHS Prostate Cancer Risk Management Programme (PCRMP) has published updated
guidelines in 2009 on how to handle requests for PSA testing in asymptomatic men. A recent
European trial (ERSPC) showed a statistically significant reduction in the rate of death prostate
cancer by 20% in men aged 55 to 69 years but this was associated with a high risk of over-diagnosis
and over-treatment. Having reviewed this and other data the National Screening Committee have
decided not to introduce a prostate cancer screening programme yet but rather allow men to make
an informed choice.
Age-adjusted upper limits for PSA were recommended by the PCRMP:

Age

PSA level (ng/ml)

50-59 years

3.0

60-69 years

4.0

> 70 years

5.0

PSA levels may also be raised by*:







benign prostatic hyperplasia (BPH)
prostatitis and urinary tract infection (NICE recommend to postpone the PSA test for at least
1 month after treatment)
ejaculation (ideally not in the previous 48 hours)
vigorous exercise (ideally not in the previous 48 hours)
urinary retention
instrumentation of the urinary tract

Poor specificity and sensitivity




around 33% of men with a PSA of 4-10 ng/ml will be found to have prostate cancer. With a
PSA of 10-20 ng/ml this rises to 60% of men
around 20% with prostate cancer have a normal PSA
various methods are used to try and add greater meaning to a PSA level including ageadjusted upper limits and monitoring change in PSA level with time (PSA velocity or PSA
doubling time)

*whether digital rectal examination actually causes a rise in PSA levels is a matter of debate

Question 42 of 69
You review a 9-month-old who has parents of Jamaican origin. His parents have noticed a small
swelling around his umbilicus. He is a well child who is on the 50th centile. On examination you note
a small, reducible umbilical hernia which is less than 1 cm in size. What is the most appropriate
management?
A. Contact the local Child Protection Officer
B. Admit to paediatrics
C. Reassure the parents that the vast majority resolve by the age of 4-5 years
D. Refer to paediatric surgeon

E. Refer to a paediatrician for a sweat test

Congenital hernias



inguinal: repair ASAP
umbilical: manage conservatively

This little boy has an umbilical hernia. The vast majority are managed conservatively as usually
(>90%) resolve spontaneously.
Abdominal wall hernias
The classical surgical definition of a hernia is the protrusion of an organ or the fascia of an organ
through the wall of the cavity that normally contains it.
Risk factors for abdominal wall hernias include:





obesity
ascites
increasing age
surgical wounds

Features


palpable lump



cough impulse
pain
obstruction: more common in femoral hernias
strangulation: may compromise the bowel blood supply leading to infarction





Types of abdominal wall hernias:

Type of hernia
Inguinal hernia

Details
Inguinal hernias account for 75% of abdominal wall hernias. Around 95% of patients are male;

Type of hernia

Details
men have around a 25% lifetime risk of developing an inguinal hernia.
Above and medial to pubic tubercle
Strangulation is rare

Femoral hernia

Below and lateral to the pubic tubercle
More common in women, particularly multiparous ones
High risk of obstruction and strangulation
Surgical repair is required

Umbilical hernia

Symmetrical bulge under the umbilicus

Paraumbilical
hernia

Asymmetrical bulge - half the sac is covered by skin of the abdomen directly above or below
the umbilicus

Epigastric hernia

Lump in the midline between umbilicus and the xiphisternum
Most common in men aged 20-30 years

Incisional hernia

May occur in up to 10% of abdominal operations

Spigelian hernia

Also known as lateral ventral hernia
Rare and seen in older patients
A hernia through the spigelian fascia (the aponeurotic layer between the rectus abdominis
muscle medially and the semilunar line laterally)

Obturator hernia

A hernia which passes through the obturator foramen. More common in females and typical
presents with bowel obstruction

Richter hernia

A rare type of hernia where only the antimesenteric border of the bowel herniates through the
fascial defect

Abdominal wall hernias in children:

Type of hernia

Details

Congenital inguinal hernia

Indirect hernias resulting from a patent processus vaginalis
Occur in around 1% of term babies. More common in premature babies and boys
60% are right sided, 10% are bilaterally
Should be surgically repaired soon after diagnosis as at risk of incarceration

Infantile umbilical hernia

Symmetrical bulge under the umbilicus
More common in premature and Afro-Caribbean babies

Type of hernia

Details
The vast majority resolve without intervention before the age of 4-5 years
Complications are rare

Question 43 of 69
Which one of the following statements regarding lidocaine is correct?
A. Preparations mixed with adrenaline should not be used for minor surgery
involving the finger
B. The maximum dose of lidocaine is 5mg/kg
C. The anaesthetic effect usual wears off after 15-20 minutes
D. Is contraindicated in patients with a history of ventricular tachycardia
E. Preparations mixed with adrenaline are more likely to cause blood loss

Minor surgery
Local anaesthetic (LA)
Lidocaine is the most widely used LA. It has a rapid onset of action and anaesthesia lasts for around
1 hour.




the maximum safe dose is 3mg/kg. The BNF states 200mg (or 500mg if given in solutions
containing adrenaline), which equates to 3mg/kg for a 66kg patient. This is the equivalent of
20ml of 1% solution or 10ml of 2% solution
lidocaine is available pre-mixed with adrenaline. This increases the duration of action of
lidocaine and reduces blood loss secondary to vasoconstriction. It must never be used near
extremities due to the risk of ischaemia

Suture material

Non-absorbable Absorbable
Silk
Novafil
Prolene
Ethilon

Vicryl
Dexon
PDS

Non-absorbable sutures are normally removed after 7-14 days, depending on the location.
Absorbable sutures normally disappear after 7-10 days. Removal times for non-absorbable sutures
are shown below:

Area

Removal time (days)

Face

3-5

Scalp, limbs, chest

7 - 10

Hand, foot, back

10 - 14

Question 44 of 69
A 62-year-old man is called for review after a positive faecal occult blood test done as part of the
national screening programme. During counselling for colonoscopy he asks what percentage of
patients with a positive faecal occult blood test have colorectal cancer. What is the most accurate
answer?
A. 0.5 - 2%
B. 5 - 15%
C. 20 - 30%
D. 30 - 50%
E. 55 - 75%

Colorectal cancer screening - PPV of FOB = 5 - 15%

There is also a 30-45% chance of having an adenoma with a positive faecal occult blood test
Colorectal cancer: screening
Overview





most cancers develop from adenomatous polyps. Screening for colorectal cancer has been
shown to reduce mortality by 16%
the NHS now has a national screening programme offering screening every 2 years to all
men and women aged 60 to 69 years. Patients aged over 70 years may request screening
eligible patients are sent faecal occult blood (FOB) tests through the post
patients with abnormal results are offered a colonoscopy

At colonoscopy, approximately:




5 out of 10 patients will have a normal exam
4 out of 10 patients will be found to have polyps which may be removed due to their
premalignant potential
1 out of 10 patients will be found to have cancer

Question 45 of 69
Which one of the following statements regarding male circumcision is correct?
A. Circumcision should always be performed under a general anaesthetic
B. It is available on the NHS in areas with a high Jewish or Islamic population
C. Increases the risk of penile cancer
D. Reduces the rate of HIV transmission
E. All infants with hypospadias should be circumcised before the age of 1 year

Circumcision
Circumcision has been performed in a variety of cultures for thousands of years. Today it is mainly
people of the Jewish and Islamic faith who undergo circumcision for religious/cultural reasons.
Circumcision for religious or cultural reasons is not available on the NHS.
The medical benefits of routine circumcision remain controversial although some evidence has
emerged that it:


reduces the risk of penile cancer



reduces the risk of UTI
reduces the risk of acquiring sexually transmitted infections including HIV



Medical indications for circumcision





phimosis
recurrent balanitis
balanitis xerotica obliterans
paraphimosis

It is important to exclude hypospadias prior to circumcision as the foreskin may be used in surgical
repair. Circumcision may be performed under a local or general anaesthetic.

Question 46 of 69
A 72-year-old man is diagnosed with prostate cancer and goserelin (Zoladex) is prescribed. Which
one of the following is it most important to co-prescribe for the first three weeks of treatment?
A. Tamoxifen
B. Lansoprazole
C. Allopurinol
D. Cyproterone acetate
E. Tamsulosin

Anti-androgen treatment such as cyproterone acetate should be co-prescribed when starting
gonadorelin analogues due to the risk of tumour flare. This phenomenon is secondary to initial
stimulation of luteinising hormone release by the pituitary gland resulting in increased testosterone
levels.
The BNF advises starting cyproterone acetate 3 days before the gonadorelin analogue.
Prostate cancer: management
Localised prostate cancer (T1/T2)
Treatment depends on life expectancy and patient choice. Options include:




conservative: active monitoring & watchful waiting
radical prostatectomy
radiotherapy: external beam and brachytherapy

Localised advanced prostate cancer (T3/T4)
Options include:




hormonal therapy: see below
radical prostatectomy
radiotherapy: external beam and brachytherapy

Metastatic prostate cancer disease - hormonal therapy
Synthetic GnRH agonist



e.g. Goserelin (Zoladex)
cover initially with anti-androgen to prevent rise in testosterone

Anti-androgen


cyproterone acetate prevents DHT binding from intracytoplasmic protein complexes

Orchidectomy

Question 47 of 69
A 64-year-old man who is asymptomatic requests a PSA test. What is the upper limit of normal for a
man of this age?
A. 3.0 ng/ml
B. 3.5 ng/ml
C. 4.0 ng/ml
D. 4.5 ng/ml
E. 5.0 ng/ml

Prostate cancer: PSA testing
Prostate specific antigen (PSA) is a serine protease enzyme produced by normal and malignant
prostate epithelial cells. It has become an important tumour marker but much controversy still exists
regarding its usefulness as a screening tool.
The NHS Prostate Cancer Risk Management Programme (PCRMP) has published updated
guidelines in 2009 on how to handle requests for PSA testing in asymptomatic men. A recent
European trial (ERSPC) showed a statistically significant reduction in the rate of death prostate
cancer by 20% in men aged 55 to 69 years but this was associated with a high risk of over-diagnosis
and over-treatment. Having reviewed this and other data the National Screening Committee have
decided not to introduce a prostate cancer screening programme yet but rather allow men to make
an informed choice.
Age-adjusted upper limits for PSA were recommended by the PCRMP:

Age

PSA level (ng/ml)

50-59 years

3.0

60-69 years

4.0

> 70 years

5.0

PSA levels may also be raised by*:







benign prostatic hyperplasia (BPH)
prostatitis and urinary tract infection (NICE recommend to postpone the PSA test for at least
1 month after treatment)
ejaculation (ideally not in the previous 48 hours)
vigorous exercise (ideally not in the previous 48 hours)
urinary retention
instrumentation of the urinary tract

Poor specificity and sensitivity




around 33% of men with a PSA of 4-10 ng/ml will be found to have prostate cancer. With a
PSA of 10-20 ng/ml this rises to 60% of men
around 20% with prostate cancer have a normal PSA
various methods are used to try and add greater meaning to a PSA level including ageadjusted upper limits and monitoring change in PSA level with time (PSA velocity or PSA
doubling time)

*whether digital rectal examination actually causes a rise in PSA levels is a matter of debate

Question 48 of 69
A 33-year-old pregnant woman presents with pruritus ani. Which one of the following statements
regarding haemorrhoids is incorrect?
A. Painless rectal bleeding is the most common symptom
B. Haemorrhoidal tissue is part of the normal anatomy
C. Internal haemorrhoids do not generally cause pain
D. Soiling may be seen
E. Usually occur at the 1 o'clock, 5 o'clock and 9 o'clock position

Haemorrhoids usually occur at the 3 o'clock, 7 o'clock and 11 o'clock position
Haemorrhoids
Haemorrhoidal tissue is part of the normal anatomy which contributes to anal continence. These
mucosal vascular cushions are found in the left lateral, right posterior and right anterior portions of
the anal canal (3 o'clock, 7'o'clock and 11 o'clock respectively). Haemorrhoids are said to exist when
they become enlarged, congested and symptomatic
Clinical features





painless rectal bleeding is the most common symptom
pruritus
pain: usually not significant unless piles are thrombosed
soiling may occur with third or forth degree piles

Types of haemorrhoids
External



originate below the dentate line
prone to thrombosis, may be painful

Internal



originate above the dentate line
do not generally cause pain

Grading of internal haemorrhoids

Grade I

Do not prolapse out of the anal canal

Grade II

Prolapse on defecation but reduce spontaneously

Grade III

Can be manually reduced

Grade IV

Cannot be reduced

Management






soften stools: increase dietary fibre and fluid intake
topical local anaesthetics and steroids may be used to help symptoms
outpatient treatments: rubber band ligation is superior to injection sclerotherapy
surgery is reserved for large symptomatic haemorrhoids which do not respond to outpatient
treatments
newer treatments: Doppler guided haemorrhoidal artery ligation, stapled haemorrhoidopexy

Acutely thrombosed external haemorrhoids




typically present with significant pain
examination reveals a purplish, oedematous, tender subcutaneous perianal mass
if patient presents within 72 hours then referral should be considered for excision. Otherwise
patients can usually be managed with stool softeners, ice packs and analgesia. Symptoms
usually settle within 10 days

Question 49 of 69
A 76-year-old man presents with lower urinary tract symptoms. Following a digital rectal examination
and prostate specific antigen test a diagnosis of benign prostatic hyperplasia is made and finasteride
is started. What is the mechanism of action of this drug?
A. Alpha-1 antagonist
B. 5-alpha receptor antagonist
C. Testosterone receptor antagonist
D. Alpha-1 agonists
E. Inhibits conversion of testosterone to dihydrotestosterone

Finasteride: 5 alpha-reductase inhibitor - inhibits conversion of testosterone to dihydrotestosterone

Benign prostatic hyperplasia
Benign prostatic hyperplasia (BPH) is a common condition seen in older men.
Risk factors



age: around 50% of 50-year-old men will have evidence of BPH and 30% will have
symptoms. Around 80% of 80-year-old men have evidence of BPH
ethnicity: black > white > Asian

BPH typically presents with lower urinary tract symptoms (LUTS), which may be categorised into:





voiding symptoms (obstructive): weak or intermittent urinary flow, straining, hesitancy,
terminal dribbling and incomplete emptying
storage symptoms (irritative) urgency, frequency, urgency incontinence and nocturia
post-micturition: dribbling
complications: urinary tract infection, retention, obstructive uropathy

Management options




watchful waiting
medication: alpha-1 antagonists, 5 alpha-reductase inhibitors. The use of combination
therapy was supported by the Medical Therapy Of Prostatic Symptoms (MTOPS) trial
surgery: transurethral resection of prostate (TURP)

Alpha-1 antagonists e.g. tamsulosin, alfuzosin




decrease smooth muscle tone (prostate and bladder)
considered first-line, improve symptoms in around 70% of men
adverse effects: dizziness, postural hypotension, dry mouth, depression

5 alpha-reductase inhibitors e.g. finasteride





block the conversion of testosterone to dihydrotestosterone (DHT), which is known to induce
BPH
unlike alpha-1 antagonists causes a reduction in prostate volume and hence may slow
disease progression. This however takes time and symptoms may not improve for 6 months.
They may also decrease PSA concentrations by up to 50%
adverse effects: erectile dysfunction, reduced libido, ejaculation problems, gynaecomastia

Question 50 of 69
What is the lifetime risk of developing colorectal cancer in the United Kingdom?
A. 1%
B. 2%
C. 5%
D. 10%
E. 15%
Next question
Colorectal cancer is the third most common cancer in the UK, with approximately 30,000 new cases
in England and Wales per year
Colorectal cancer: screening
Overview





most cancers develop from adenomatous polyps. Screening for colorectal cancer has been
shown to reduce mortality by 16%
the NHS now has a national screening programme offering screening every 2 years to all
men and women aged 60 to 69 years. Patients aged over 70 years may request screening
eligible patients are sent faecal occult blood (FOB) tests through the post
patients with abnormal results are offered a colonoscopy

At colonoscopy, approximately:




5 out of 10 patients will have a normal exam
4 out of 10 patients will be found to have polyps which may be removed due to their
premalignant potential
1 out of 10 patients will be found to have cancer

Question 51 of 69
What is the maximum safe volume of lidocaine 1% that may be used during minor surgery on an
average sized adult?
A. 10 ml
B. 30 ml
C. 50 ml
D. 20 ml
E. 5 ml

Minor surgery
Local anaesthetic (LA)
Lidocaine is the most widely used LA. It has a rapid onset of action and anaesthesia lasts for around
1 hour.




the maximum safe dose is 3mg/kg. The BNF states 200mg (or 500mg if given in solutions
containing adrenaline), which equates to 3mg/kg for a 66kg patient. This is the equivalent of
20ml of 1% solution or 10ml of 2% solution
lidocaine is available pre-mixed with adrenaline. This increases the duration of action of
lidocaine and reduces blood loss secondary to vasoconstriction. It must never be used near

extremities due to the risk of ischaemia
Suture material

Non-absorbable Absorbable
Silk
Novafil
Prolene
Ethilon

Vicryl
Dexon
PDS

Non-absorbable sutures are normally removed after 7-14 days, depending on the location.
Absorbable sutures normally disappear after 7-10 days. Removal times for non-absorbable sutures
are shown below:

Area

Removal time (days)

Face

3-5

Scalp, limbs, chest

7 - 10

Hand, foot, back

10 - 14

Question 52 of 69

Next

A 50-year-old woman presents with right-sided medial thigh pain for the past week. There has been
no change in her bowels. On examination you noticed a grape sized lump below and lateral to the
right pubic tubercle which is difficult to reduce. What is the most likely diagnosis?
A. Inguinal hernia
B. Richter hernia
C. Spigelian hernia
D. Obturator hernia
E. Femoral hernia

Abdominal wall hernias
The classical surgical definition of a hernia is the protrusion of an organ or the fascia of an organ
through the wall of the cavity that normally contains it.
Risk factors for abdominal wall hernias include:





obesity
ascites
increasing age
surgical wounds

Features


palpable lump



cough impulse
pain
obstruction: more common in femoral hernias
strangulation: may compromise the bowel blood supply leading to infarction





Types of abdominal wall hernias:

Type of hernia

Details

Inguinal hernia

Inguinal hernias account for 75% of abdominal wall hernias. Around 95% of patients are male;
men have around a 25% lifetime risk of developing an inguinal hernia.
Above and medial to pubic tubercle
Strangulation is rare

Femoral hernia

Below and lateral to the pubic tubercle
More common in women, particularly multiparous ones
High risk of obstruction and strangulation
Surgical repair is required

Umbilical hernia

Symmetrical bulge under the umbilicus

Paraumbilical
hernia

Asymmetrical bulge - half the sac is covered by skin of the abdomen directly above or below
the umbilicus

Epigastric hernia

Lump in the midline between umbilicus and the xiphisternum

Type of hernia

Details
Most common in men aged 20-30 years

Incisional hernia

May occur in up to 10% of abdominal operations

Spigelian hernia

Also known as lateral ventral hernia
Rare and seen in older patients
A hernia through the spigelian fascia (the aponeurotic layer between the rectus abdominis
muscle medially and the semilunar line laterally)

Obturator hernia

A hernia which passes through the obturator foramen. More common in females and typical
presents with bowel obstruction

Richter hernia

A rare type of hernia where only the antimesenteric border of the bowel herniates through the
fascial defect

Abdominal wall hernias in children:

Type of hernia

Details

Congenital inguinal hernia

Indirect hernias resulting from a patent processus vaginalis
Occur in around 1% of term babies. More common in premature babies and boys
60% are right sided, 10% are bilaterally
Should be surgically repaired soon after diagnosis as at risk of incarceration

Infantile umbilical hernia

Symmetrical bulge under the umbilicus
More common in premature and Afro-Caribbean babies
The vast majority resolve without intervention before the age of 4-5 years
Complications are rare

Question 53-55 of 69
Theme: Suture removal
A. 2 days
B. 4 days
C. 8 days
D. 12 days
E. 16 days
F. 21 days
For each one of the following locations please select the optimal time to remove the sutures.
Assume the patient has had a small skin lesion removed in primary care and has no relevant
medical history.

53.

Back
12 days

54.

Face
4 days

55.

Scalp
8 days

Minor surgery

Local anaesthetic (LA)
Lidocaine is the most widely used LA. It has a rapid onset of action and anaesthesia lasts for around
1 hour.



the maximum safe dose is 3mg/kg. The BNF states 200mg (or 500mg if given in solutions



containing adrenaline), which equates to 3mg/kg for a 66kg patient. This is the equivalent of
20ml of 1% solution or 10ml of 2% solution
lidocaine is available pre-mixed with adrenaline. This increases the duration of action of
lidocaine and reduces blood loss secondary to vasoconstriction. It must never be used near
extremities due to the risk of ischaemia

Suture material

Non-absorbable Absorbable
Silk
Novafil
Prolene
Ethilon

Vicryl
Dexon
PDS

Non-absorbable sutures are normally removed after 7-14 days, depending on the location.
Absorbable sutures normally disappear after 7-10 days. Removal times for non-absorbable sutures
are shown below:

Area

Removal time (days)

Face

3-5

Scalp, limbs, chest

7 - 10

Hand, foot, back

10 - 14

Question 56 of 69
A 31-year-old man returns for review. He was diagnosed with an anal fissure around 7 weeks ago
and has tried dietary modification, laxatives and topical anaesthetic with little benefit. What is the
most appropriate next step?
A. Refer to secondary care
B. Oral calcium channel blocker
C. Topical steroid
D. Buccal glyceryl trinitrate prior to defecation
E. Topical glyceryl trinitrate

Anal fissure - topical glyceryl trinitrate

Anal fissure
Anal fissures are longitudinal or elliptical tears of the squamous lining of the distal anal canal. If
present for less than 6 weeks they are defined as acute, and chronic if present for more than 6
weeks. Around 90% of anal fissures occur on the posterior midline
Management of an acute anal fissure (< 6 weeks)





dietary advice: high-fibre diet with high fluid intake
bulk-forming laxatives are first line - if not tolerated then lactulose should be tried
lubricants such as petroleum jelly may be tried before defecation
topical anaesthetics

-analgesia


topical steroids do not provide significant relief

Management of a chronic anal fissure (> 6 weeks)




the above techniques should be continued
topical glyceryl trinitrate (GTN) is first line treatment for a chronic anal fissure
if topical GTN is not effective after 8 weeks then secondary referral should be considered for
surgery or botulinum toxin

Question 57 of 69
A 50-year-old man who is known to have obesity and hypertension comes for review. His current
BMI is 38 kg/m^2 and blood pressure today is 154/92 mmHg despite ramipril and
bendroflumethiazide. Lifestyle and a trial of orlistat have failed to reduce his weight. Which one of
the following is the most suitable intervention?
A. Biliopancreatic diversion with duodenal switch
B. Laparoscopic-adjustable gastric banding
C. Trial of sibutramine
D. Referral for counselling to discuss his excessive eating
E. Sleeve gastrectomy

Laparoscopic-adjustable gastric banding is the intervention of choice in patients with a BMI < 40
kg/m^2.
Sibutramine has recently been withdrawn due to concerns about a possible increased risk of
cardiovascular events.
Obesity: bariatric surgery
The use of bariatric surgery in the management of obesity has developed significantly over the past
decade. It is now recognised that for many obese patients who fail to lose weight with lifestyle and

drug interventions the risks and expense of long-term obesity outweigh those of surgery.
NICE guidelines on bariatric surgery for adults
Consider surgery for people with severe obesity if:







they have a BMI of 40 kg/m^2 or more, or between 35 kg/m^2 and 40 kg/m^2 and other
significant disease (for example, type 2 diabetes mellitus, hypertension) that could be
improved if they lost weight
all appropriate non-surgical measures have failed to achieve or maintain adequate clinically
beneficial weight loss for at least 6 months
they are receiving or will receive intensive specialist management
they are generally fit for anaesthesia and surgery
they commit to the need for long-term follow-up

Consider surgery as a first-line option for adults with a BMI of more than 50 kg/m2 in whom surgical
intervention is considered appropriate; consider orlistat before surgery if the waiting time is long
Types of bariatric surgery:


primarily restrictive: laparoscopic-adjustable gastric banding (LAGB) or sleeve gastrectomy



primarily malabsorptive: classic biliopancreatic diversion (BPD) has now largely been
replaced by biliopancreatic diversion with duodenal switch
mixed: Roux-en-Y gastric bypass surgery



Which operation?





LAGB produces less weight loss than malabsorptive or mixed procedures but as it has fewer
complications it is normally the first-line intervention in patients with a BMI of 30-39kg/m^2
patients with a BMI > 40 kg/m^2 may be considered for a gastric bypass or sleeve
gastrectomy. The latter may be done as a sole procedure or as an initial procedure prior to
bypass
primarily malabsorptive procedures are usually reserved for very obese patients (e.g. BMI >
60 kg/m^2)

Question 58 of 69
Which one of the following statements regarding congenital inguinal hernias is correct?
A. They should be managed conservatively
B. Result from the premature closure of the processus vaginalis
C. They are more common in girls
D. The incidence in newborns is 0.1-0.2%
E. They are more common on the right side

Abdominal wall hernias
The classical surgical definition of a hernia is the protrusion of an organ or the fascia of an organ
through the wall of the cavity that normally contains it.
Risk factors for abdominal wall hernias include:





obesity
ascites
increasing age
surgical wounds

Features






palpable lump
cough impulse
pain
obstruction: more common in femoral hernias
strangulation: may compromise the bowel blood supply leading to infarction

Types of abdominal wall hernias:

Type of hernia

Details

Inguinal hernia

Inguinal hernias account for 75% of abdominal wall hernias. Around 95% of patients are male;
men have around a 25% lifetime risk of developing an inguinal hernia.
Above and medial to pubic tubercle
Strangulation is rare

Femoral hernia

Below and lateral to the pubic tubercle
More common in women, particularly multiparous ones
High risk of obstruction and strangulation
Surgical repair is required

Umbilical hernia

Symmetrical bulge under the umbilicus

Paraumbilical
hernia

Asymmetrical bulge - half the sac is covered by skin of the abdomen directly above or below
the umbilicus

Epigastric hernia

Lump in the midline between umbilicus and the xiphisternum
Most common in men aged 20-30 years

Incisional hernia

May occur in up to 10% of abdominal operations

Spigelian hernia

Also known as lateral ventral hernia
Rare and seen in older patients
A hernia through the spigelian fascia (the aponeurotic layer between the rectus abdominis
muscle medially and the semilunar line laterally)

Obturator hernia

A hernia which passes through the obturator foramen. More common in females and typical
presents with bowel obstruction

Richter hernia

A rare type of hernia where only the antimesenteric border of the bowel herniates through the
fascial defect

Abdominal wall hernias in children:

Type of hernia

Details

Congenital inguinal hernia

Indirect hernias resulting from a patent processus vaginalis
Occur in around 1% of term babies. More common in premature babies and boys
60% are right sided, 10% are bilaterally
Should be surgically repaired soon after diagnosis as at risk of incarceration

Infantile umbilical hernia

Symmetrical bulge under the umbilicus

Type of hernia

Details
More common in premature and Afro-Caribbean babies
The vast majority resolve without intervention before the age of 4-5 years
Complications are rare

Question 59-61 of 69
Theme: Breast disorders
A. Lipoma
B. Paget's disease of the breast
C. Breast cancer
D. Sebaceous cysts
E. Fibroadenoma
F. Fibroadenosis
G. Duct papilloma
H. Breast abscess
I. Fat necrosis
J. Mammary duct ectasia
For each one of the following please select the most appropriate answer:

59.

A 49-year-old woman presents with a tender lump around the areola associated with a green nipple
discharge.

The correct answer is Mammary duct ectasia

60.

An obese woman presents with an irregular lump on the lateral aspect of her right breast associated with
skin tethering. Biopsy excludes a malignant cause.

The correct answer is Fat necrosis

61.

A 41-year-old woman presents with pain and an irregular mobile lump in her left breast.

The correct answer is Breast cancer

Breast disorders

The table below describes some of the features seen in the most common breast disorders:

Disorder

Features

Fibroadenoma

Common in women under the age of 30 years
Often described as 'breast mice' due as they are discrete, non-tender, highly
mobile lumps

Fibroadenosis (fibrocystic disease,
benign mammary dysplasia)

Most common in middle-aged women
'Lumpy' breasts which may be painful. Symptoms may worsen prior to
menstruation

Breast cancer

Characteristically a hard, irregular lump. There may be associated nipple
inversion or skin tethering
Paget's disease of the breast - intraductal carcinoma associated with a
reddening and thickening (may resemble eczematous changes) of the
skin/areola

Mammary duct ectasia

Dilatation of the large breast ducts
Most common around the menopause
May present with a tender lump around the areola +/- a green nipple
discharge
If ruptures may cause local inflammation, sometimes referred to as 'plasma
cell mastitis'

Duct papilloma

Local areas of epithelial proliferation in large mammary ducts
Hyperplastic lesions rather than malignant or premalignant
May present with blood stained discharge

Disorder

Features

Fat necrosis

More common in obese women with large breasts
May follow trivial or unnoticed trauma
Initial inflammatory response, the lesion is typical firm and round but may
develop into a hard, irregular breast lump
Rare and may mimic breast cancer so further investigation is always
warranted

Breast abscess

More common in lactating women
Red, hot tender swelling

Lipomas and sebaceous cysts may also develop around the breast tissue.

Question 62 of 69
Which one of the following statements regarding testicular cancer is correct?
A. Fragile X syndrome is a risk factor
B. Gynaecomastia is seen in the majority of men
C. Seminomas have a better prognosis than teratomas
D. Afro-Caribbean ethnicity is a risk factor
E. May present as a varicocele in up to 10% of patients

Testicular cancer
Testicular cancer is the most common malignancy in men aged 20-30 years. Around 95% of cases
of testicular cancer are germ-cell tumours. Germ cell tumours may essentially be divided into:



seminomas
teratomas

Other type of germ cell tumours include yolk sac tumours. Non-germ cell tumours include Leydig cell
tumours and sarcomas.
The peak incidence for teratomas is 25 years and seminomas is 35 years. Risk factors include:






cryptorchidism
infertility
family history
Klinefelter's syndrome
mumps orchitis

Features


a painless lump is the most common presenting symptom



pain may also be present in a minority of men
other possible features include hydrocele, gynaecomastia



Diagnosis


ultrasound is first-line

Management



orchidectomy
chemotherapy and radiotherapy may be given depending on staging

Prognosis is generally excellent



5 year survival for seminomas is around 95% if Stage I
5 year survival for teratomas is around 85% if Stage I

Question 63 of 69
You are discussing an elevated PSA result with one of your patients, a 62-year-old man with a PSA
level of 7.2 ng/ml. Which procedure is he most likely to have following referral to a urologist?
A. Prostatectomy
B. Cystoscopy with prostate biopsy
C. Staging CT scan
D. MRI pelvis
E. TRUS-guided biopsy

A TRUS-guided biopsy is need to clarify the diagnosis as around two-thirds of such patients will not
have prostate cancer.
Prostate cancer: PSA testing
Prostate specific antigen (PSA) is a serine protease enzyme produced by normal and malignant
prostate epithelial cells. It has become an important tumour marker but much controversy still exists
regarding its usefulness as a screening tool.
The NHS Prostate Cancer Risk Management Programme (PCRMP) has published updated
guidelines in 2009 on how to handle requests for PSA testing in asymptomatic men. A recent
European trial (ERSPC) showed a statistically significant reduction in the rate of death prostate
cancer by 20% in men aged 55 to 69 years but this was associated with a high risk of over-diagnosis
and over-treatment. Having reviewed this and other data the National Screening Committee have
decided not to introduce a prostate cancer screening programme yet but rather allow men to make
an informed choice.
Age-adjusted upper limits for PSA were recommended by the PCRMP:

Age

PSA level (ng/ml)

50-59 years

3.0

60-69 years

4.0

Age

PSA level (ng/ml)

> 70 years

5.0

PSA levels may also be raised by*:


benign prostatic hyperplasia (BPH)



prostatitis and urinary tract infection (NICE recommend to postpone the PSA test for at least
1 month after treatment)



ejaculation (ideally not in the previous 48 hours)
vigorous exercise (ideally not in the previous 48 hours)
urinary retention
instrumentation of the urinary tract
Poor specificity and sensitivity









around 33% of men with a PSA of 4-10 ng/ml will be found to have prostate cancer. With a
PSA of 10-20 ng/ml this rises to 60% of men
around 20% with prostate cancer have a normal PSA
various methods are used to try and add greater meaning to a PSA level including ageadjusted upper limits and monitoring change in PSA level with time (PSA velocity or PSA
doubling time)
*whether digital rectal examination actually causes a rise in PSA levels is a matter of debate

Question 64 of 69
A 34-year-old man presents to an emergency surgery with abdominal pain. This started earlier on in
the day and is getting progressively worse. The pain is located on his left flank and radiates down
into his groin. He has had no similar pain previously and is normally fit and well. Examination reveals
a man who is flushed and sweaty but is otherwise unremarkable. What is the most suitable initial
management?
A. Oral ciprofloxacin
B. IM diclofenac 75 mg
C. Oral co-amoxiclav and metronidazole
D. IM morphine 5 mg
E. IM diclofenac 75 mg + start bendroflumethiazide to prevent further episodes

This man may need to be referred acutely to the surgeons for pain relief and investigations to
exclude obstruction. It would not be suitable to start bendroflumethiazide in the initial phase of the
first episode
Renal stones: management
Acute management of renal colic
Medication


the British Association of Urological Surgeons (BAUS) recommend diclofenac



(intramuscular/oral) as the analgesia of choice for renal colic*
BAUS also endorse the widespread use of alpha-adrenergic blockers to aid ureteric stone
passage

Imaging



patients presenting to the Emergency Department usually have a KUB x-ray (shows 60% of
stones)
the imaging of choice is a non-contrast CT (NCCT). 99% of stones are identifiable on NCCT.
Many GPs now have direct access to NCCT

Stones < 5 mm will usually pass spontaneously. Lithotripsy and nephrolithotomy may be for severe
cases.
Prevention of renal stones
Calcium stones may be due to hypercalciuria, which is found in up to 5-10% of the general
population.




high fluid intake
low animal protein, low salt diet (a low calcium diet has not been shown to be superior to a
normocalcaemic diet)
thiazides diuretics (increase distal tubular calcium resorption)

Oxalate stones


cholestyramine reduces urinary oxalate secretion



pyridoxine reduces urinary oxalate secretion

Uric acid stones



allopurinol
urinary alkalinization e.g. oral bicarbonate

*Diclofenac use is now less common following the MHRA warnings about cardiovascular risk. It is
therefore likely the guidelines will change soon to an alternative NSAID such as naproxen

Question 65 of 69
Which one of the following statements regarding prostate specific antigen (PSA) testing is NOT true?
A. Around a third of men with a PSA of 4-10 ng/ml will be found to have prostate
cancer
B. A PSA level of 3.8 ng/ml in a 55-year-old man is normal
C. Around 20% of patients diagnosed with prostate cancer have a normal PSA
D. PSA levels rise following ejaculation
E. Very high PSA levels (e.g. > 50 ng/ml) suggest metastatic disease

Prostate cancer: PSA testing
Prostate specific antigen (PSA) is a serine protease enzyme produced by normal and malignant
prostate epithelial cells. It has become an important tumour marker but much controversy still exists
regarding its usefulness as a screening tool.

The NHS Prostate Cancer Risk Management Programme (PCRMP) has published updated
guidelines in 2009 on how to handle requests for PSA testing in asymptomatic men. A recent
European trial (ERSPC) showed a statistically significant reduction in the rate of death prostate
cancer by 20% in men aged 55 to 69 years but this was associated with a high risk of over-diagnosis
and over-treatment. Having reviewed this and other data the National Screening Committee have
decided not to introduce a prostate cancer screening programme yet but rather allow men to make
an informed choice.
Age-adjusted upper limits for PSA were recommended by the PCRMP:

Age

PSA level (ng/ml)

50-59 years

3.0

60-69 years

4.0

> 70 years

5.0

PSA levels may also be raised by*:







benign prostatic hyperplasia (BPH)
prostatitis and urinary tract infection (NICE recommend to postpone the PSA test for at least
1 month after treatment)
ejaculation (ideally not in the previous 48 hours)
vigorous exercise (ideally not in the previous 48 hours)
urinary retention
instrumentation of the urinary tract

Poor specificity and sensitivity




around 33% of men with a PSA of 4-10 ng/ml will be found to have prostate cancer. With a
PSA of 10-20 ng/ml this rises to 60% of men
around 20% with prostate cancer have a normal PSA
various methods are used to try and add greater meaning to a PSA level including ageadjusted upper limits and monitoring change in PSA level with time (PSA velocity or PSA
doubling time)

*whether digital rectal examination actually causes a rise in PSA levels is a matter of debate

Question 66 of 69
A 79-year-old complains of lower urinary tract symptoms. Which one of the following statements
regarding benign prostatic hyperplasia is incorrect?
A. Goserelin is licensed for refractory cases
B. Side-effects of 5 alpha-reductase inhibitors include ejaculation disorders and
gynaecomastia
C. Possible presentations include recurrent urinary tract infection
D. 5 alpha-reductase inhibitors typically decrease the prostate specific antigen
level
E. More common in black men

Goserelin (Zoladex) is not used in the management of benign prostatic hyperplasia
Benign prostatic hyperplasia
Benign prostatic hyperplasia (BPH) is a common condition seen in older men.
Risk factors



age: around 50% of 50-year-old men will have evidence of BPH and 30% will have
symptoms. Around 80% of 80-year-old men have evidence of BPH
ethnicity: black > white > Asian

BPH typically presents with lower urinary tract symptoms (LUTS), which may be categorised into:





voiding symptoms (obstructive): weak or intermittent urinary flow, straining, hesitancy,
terminal dribbling and incomplete emptying
storage symptoms (irritative) urgency, frequency, urgency incontinence and nocturia
post-micturition: dribbling
complications: urinary tract infection, retention, obstructive uropathy

Management options



watchful waiting



medication: alpha-1 antagonists, 5 alpha-reductase inhibitors. The use of combination
therapy was supported by the Medical Therapy Of Prostatic Symptoms (MTOPS) trial
surgery: transurethral resection of prostate (TURP)



Alpha-1 antagonists e.g. tamsulosin, alfuzosin


decrease smooth muscle tone (prostate and bladder)



considered first-line, improve symptoms in around 70% of men
adverse effects: dizziness, postural hypotension, dry mouth, depression



5 alpha-reductase inhibitors e.g. finasteride





block the conversion of testosterone to dihydrotestosterone (DHT), which is known to induce
BPH
unlike alpha-1 antagonists causes a reduction in prostate volume and hence may slow
disease progression. This however takes time and symptoms may not improve for 6 months.
They may also decrease PSA concentrations by up to 50%
adverse effects: erectile dysfunction, reduced libido, ejaculation problems, gynaecomastia
Question 67 of 69

You receive a fax through from urology. One of your patients with a raised PSA recently underwent a
prostatic biopsy. The report reads as follows:

Adenocarcinoma prostate, Gleason 3+4
Which one of the following statements regarding the Gleason score is incorrect?
A. Grades the glandular architecture seen on histology following hollow needle
biopsy
B. The Gleason grade ranges from 1 to 5
C. The Gleason score ranges from 2 to 10
D. The lower the Gleason score the worse the prognosis
E. Used to predict prognosis in patients with prostatic cancer

Prostate cancer: prognosis
The Gleason score is used to predict prognosis in patients with prostatic cancer. The grading system
is based on the glandular architecture seen on histology following hollow needle biopsy
The most prevalent and the second most prevalent pattern seen are added to obtain a Gleason
score. The Gleason grade ranges from 1 to 5 meaning the Gleason score ranges from 2 to 10 (i.e.
two values added)
The higher the Gleason score the worse the prognosis

Question 68 of 69
Each one of the following is a risk factor for gastric cancer, except:
A. Blood group A
B. Pernicious anaemia
C. H. pylori infection
D. Smoking
E. History of duodenal ulceration

Gastric cancer
Epidemiology





overall incidence is decreasing, but incidence of tumours arising from the cardia is increasing
peak age = 70-80 years
more common in Japan, China, Finland and Colombia than the West
more common in males, 2:1

Associations








H. pylori infection
blood group A: gAstric cAncer
gastric adenomatous polyps
pernicious anaemia
smoking
diet: salty, spicy, nitrates
may be negatively associated with duodenal ulcer

Investigation



diagnosis: endoscopy with biopsy
staging: CT or endoscopic ultrasound - endoscopic ultrasound has recently been shown to
be superior to CT

Question 69 of 69
A 60-year-old man is worried about his risk of developing colorectal cancer. Following the
introduction of the national screening programme how often is such a patient offered a faecal occult
blood screening test?
A. Every year
B. Every two years
C. Every three years
D. Every five years
E. On one occasion at the age of 65

Colorectal cancer: screening
Overview





most cancers develop from adenomatous polyps. Screening for colorectal cancer has been
shown to reduce mortality by 16%
the NHS now has a national screening programme offering screening every 2 years to all
men and women aged 60 to 69 years. Patients aged over 70 years may request screening
eligible patients are sent faecal occult blood (FOB) tests through the post
patients with abnormal results are offered a colonoscopy

At colonoscopy, approximately:




5 out of 10 patients will have a normal exam
4 out of 10 patients will be found to have polyps which may be removed due to their
premalignant potential
1 out of 10 patients will be found to have cancer

=====================================================================================

Sponsor Documents

Or use your account on DocShare.tips

Hide

Forgot your password?

Or register your new account on DocShare.tips

Hide

Lost your password? Please enter your email address. You will receive a link to create a new password.

Back to log-in

Close